0

Mental additions and subtractions of decimals - class-VI

Description: mental additions and subtractions of decimals
Number of Questions: 43
Created by:
Tags: maths decimal representation of rational numbers and operations number system calculations and mental strategies 4 fractions and decimal numbers mental skills
Attempted 0/43 Correct 0 Score 0

Which of the following is equal to  $1?$

  1. $\dfrac { 0.304 \times 20 } { 304 \times 2 }$

  2. $\dfrac { 0.304 \times 20 } { 3 \cdot 04 \times 2 }$

  3. $\dfrac { 0.304 \times 2 } { 30 \cdot 4 \times 2 }$

  4. $\dfrac { 0.304 \times 2 } { 304 \times 0 \cdot 2 }$


Correct Option: B
Explanation:

$\begin{array}{l} \dfrac { { 0.304\times 20 } }{ { 3.04\times 2 } }  \ =\dfrac { { 304\times 20\times 100 } }{ { 304\times 2\times 1000 } }  \ =1 \ Hence, \ B\, is\, the\, correct\, answer. \end{array}$

What decimal of an hour is a second ?

  1. 0.0025

  2. 0.00027

  3. 0.0256

  4. 0.000126


Correct Option: B
Explanation:

Required decimal = $\dfrac{1}{60 \times 60}=\dfrac{1}{3600}=0.00027$

If $4.175 = \displaystyle\frac { 1 }{ 0.2395 } $, then what is $\displaystyle\frac { 1 }{ 0.0004175 } $ equal to ?

  1. 0.2395

  2. 2395

  3. 2.395

  4. 23.95


Correct Option: B
Explanation:

$\displaystyle\frac { 1 }{ 0.0004175 } = \displaystyle\frac { 1 }{ \displaystyle\frac { 4.175 }{ 10000 }  } = \displaystyle\frac { 10000 }{ 4.175 } = \displaystyle\frac { 10000 }{ \displaystyle\frac { 1 }{ 0.2395 }  } $

$= 10000 \times 0.2395 = 2395$

Evaluate : $\left( 78.34+96.68-14.44 \right) \div 4$.

  1. $34.145$

  2. $16.58$

  3. $40.145$

  4. $45.346$


Correct Option: C
Explanation:

$\left( 78.34+96.68-14.44 \right) \div 4=160.58\div 4=40.145$

$\displaystyle \frac{\left ( 0.35 \right )^{2}-\left ( 0.03 \right )^{2}}{0.19}=? $

  1. 0.32

  2. 0.48

  3. 0.76

  4. 0.64


Correct Option: D
Explanation:

$\dfrac{(0.35)^{2}-(0.03)^{2}}{0.19}$=$\dfrac{(0.35+0.03)(0.35-0.03)}{0.19}$    $[a^2-b^2=(a+b)(a-b)]$


$=\dfrac{(0.38)(0.32)}{0.19}$


$=2\times{0.32}$

$=0.64$

So the correct answer will option D

What is $\displaystyle 0.\overline{09}\times7.\overline{3}$ equal to ?

  1. $\displaystyle 0.\overline{6}$

  2. $\displaystyle 0.\overline{7}$

  3. 0.6

  4. 0.7


Correct Option: A
Explanation:

$\displaystyle 0.\overline{09}\times7.\overline{3}=\frac{9}{99}\times7\frac{3}{9}=\frac{9}{99}\times\frac{66}{9}=\frac{6}{9}=0.\overline{6}$

What is the value of $\displaystyle \left ( 4.7\times13.26+4.7\times9.43+4.7\times77.31 \right )?$

  1. $470$

  2. $235$

  3. $705$

  4. $940$


Correct Option: A
Explanation:

given that 

we have find the the value of the expression .

$4.7\times 13.26 +4.7 \times 9.43 +4.7 \times 77.31$

Taking $4.7$ common 

$= 4.7 \times [13.26 + 9.43 +77.31]$

$= 4.7 \times [100]$

$ = 470$

So option $A $ is correct


$\displaystyle \frac{\left ( 2.3 \right )^{3}-0.027}{\left ( 2.3 \right )^{2}+0.69+0.09}=? $

  1. $2.6$

  2. $2$

  3. $2.33$

  4. $2.8$


Correct Option: B
Explanation:
$\dfrac { { \left( 2.3 \right)  }^{ 3 }-0.027 }{ { \left( 2.3 \right)  }^{ 2 }+0.69+0.09 } \\ =\dfrac { 12.167-0.027 }{ 5.29+0.69+0.09 } \\ =\dfrac { 12.14 }{ 6.07 } \\ =2$
So, correct answer is option B.

$58+\cfrac{3}{100}+\cfrac{7}{1000}=..........$

  1. $58.0037$

  2. $58.37$

  3. $58.037$

  4. none of these


Correct Option: C
Explanation:

$58+\cfrac{3}{100}+\cfrac{7}{1000}$
$=58+\cfrac{0}{10}+\cfrac{3}{100}+\cfrac{7}{1000}=58.037$

Which of the following is equal to $1$?

  1. $\displaystyle \frac{(0.11)^{2}}{(1.1)^{2}\times 0.1}$

  2. $\displaystyle \frac{(1.1)^{2}}{11^{2}\times (0.01)^{2}}$

  3. $\displaystyle \frac{(0.011)^{2}}{1.1^{2}\times 0.01^2}$

  4. $\displaystyle \frac{(0.11)^{2}}{1.1^{2}\times 0.01}$


Correct Option: C
Explanation:
$\Rightarrow \cfrac{(0.11)^{2}}{(1.1)^{2}\times 0.1}=\cfrac{0.0121}{1.21\times 0.1}=\cfrac{0.0121}{0.121}=0.1$

$\Rightarrow \cfrac{(1.1)^{2}}{11^{2}\times (0.01)^{2}}=\cfrac{1.21}{121\times 0.0001}=\cfrac{0.01}{0.0001}=100$

$\Rightarrow \cfrac{(0.011)^{2}}{(1.1)^{2}\times (0.01)^{2}}=\cfrac{0.000121}{1.21\times 0.0001}=1$

$\Rightarrow \cfrac{(0.11)^{2}}{1.1^{2}\times 0.01}=\cfrac{0.0121}{1.21\times 0.01}=\cfrac{0.0121}{1.21}=1.21$

$6$ thousandths is:

  1. $0.06$

  2. $0.006$

  3. $6.000$

  4. $0.066$


Correct Option: A
Explanation:

Place value chart of decimal number

Tenths Hundredths Thousandths
10 100 1000

If $\displaystyle 1420\div 1.42 =1000,$ then $142.0\div 14.2 =$ 

  1. $1$

  2. $10$

  3. $0.10$

  4. $1000$


Correct Option: B
Explanation:

$\displaystyle \frac { 142.0 }{ 14.2 } = \frac { 142.0 }{ 14.2 }\times \frac { 10 }{ 10 } = \frac { 1420 }{ 142 }=10 $

If $2805\div 2.55=1100$, then $280.5\div 25.5= ...........$

  1. 1.1

  2. 1.01

  3. 0.11

  4. 11


Correct Option: D
Explanation:

$\frac {280.5}{25.5}=\frac {280.5}{25.5}\times \frac {10}{10}\times \frac {10}{10}$
$=\frac {2805}{2.55}\times \frac {1}{100}=\frac {1100}{100}=11$

$2\times 0.5+9\div 0.3+10\times 0.92= ...........$

  1. 33.0

  2. 40.2

  3. 6.0

  4. 31.2


Correct Option: B
Explanation:

By BODMAS rule,
$2\times 0.5+9\div 0.3+10\times 0.92$
$=2\times 0.5+30+10\times 0.92$
$=1.0+30+9.2$
$=40.2$

If $29\times 27=783$; then $0.29\times 0.27= ...............$

  1. 0.0783

  2. 0.783

  3. 78.3

  4. 7.83


Correct Option: A
Explanation:

$0.29\rightarrow 2$ decimal places
$0.27\rightarrow 2$ decimal places
$\therefore 0.29\times 0.27=0.0783$
$(2+2=4$ decimal places)

Find the value of $1000(1+0.1+0.01+0.001).$

  1. 1.111

  2. 1.11

  3. 111.1

  4. 1111


Correct Option: D
Explanation:

$1.000$
$+0.100$
$+0.010$
$\underline {+0.001}$
$\underline {1.111}$
$\Rightarrow 1.111\times 1000=1111$

The value of $\dfrac { 0.1\times 0.1\times 0.1+0.02\times 0.02\times 0.02 }{ 0.2\times 0.2\times 0.2+0.04\times 0.04\times 0.04 } $ is:

  1. $0.0125$

  2. $0.125$

  3. $0.25$

  4. $0.5$


Correct Option: B
Explanation:

Given expression $=\dfrac { { \left( 0.1 \right)  }^{ 3 }+{ \left( 0.02 \right)  }^{ 3 } }{ { 2 }^{ 3 }\left[ { \left( 0.1 \right)  }^{ 3 }+{ \left( 0.02 \right)  }^{ 3 } \right]  } =\dfrac { 1 }{ 8 } =0.125$

Evaluate : $\dfrac { { \left( 2.39 \right)  }^{ 2 }-{ \left( 1.61 \right)  }^{ 2 } }{ 2.39-1.61 } $

  1. $2$

  2. $4$

  3. $6$

  4. $8$


Correct Option: B
Explanation:

Given Expression $=\dfrac { { a }^{ 2 }-{ b }^{ 2 } }{ a-b } =\dfrac { \left( a+b \right) \left( a-b \right)  }{ \left( a-b \right)  } =\left( a+b \right) =\left( 2.39+1.61 \right) =4$

$\dfrac { \left( 0.1667 \right) \left( 0.8333 \right) \left( 0.3333 \right)  }{ \left( 0.2222 \right) \left( 0.6667 \right) \left( 0.1250 \right)  } $ is approximately equal to:

  1. $2$

  2. $2.40$

  3. $2.43$

  4. $2.50$


Correct Option: D
Explanation:

Given expression $=\dfrac { \left( 0.3333 \right)  }{ \left( 0.2222 \right)  } \times \dfrac { \left( 0.1667 \right) \left( 0.8333 \right)  }{ \left( 0.6667 \right) \left( 0.1250 \right)  } $


                             $=\dfrac { 3333 }{ 2222 } \times \dfrac { \dfrac { 1 }{ 6 } \times \dfrac { 5 }{ 6 }  }{ \dfrac { 2 }{ 3 } \times \dfrac { 125 }{ 1000 }  } $

                             $=\left( \dfrac { 3 }{ 2 } \times \dfrac { 1 }{ 6 } \times \dfrac { 5 }{ 6 } \times \dfrac { 3 }{ 2 } \times 8 \right) $

                             $=\dfrac { 5 }{ 2 } $
                             $= 2.50$

If $\dfrac { 144 }{ 0.144 } =\dfrac { 14.4 }{ x } $, then the value of $x$ is:

  1. $0.0144$

  2. $1.44$

  3. $14.4$

  4. $144$


Correct Option: A
Explanation:

$\dfrac { 144 }{ 0.144 } =\dfrac { 14.4 }{ x } $

$\Rightarrow \dfrac { 144\times 1000 }{ 144 } =\dfrac { 14.4 }{ x } $

$\Rightarrow x=\dfrac { 14.4 }{ 1000 } =0.0144$

$0.75$ of a number is $1200$. What is $\displaystyle\frac{5}{8}$ of that number?

  1. $1000$

  2. $1060$

  3. $880$

  4. $8002$


Correct Option: A
Explanation:

Let the required number be $x$
According to question, we have
$0.75$ of $x$ $=1200$
$\Rightarrow \displaystyle\frac{75}{100}\times x=1200$
$\Rightarrow \displaystyle x=1200\times \frac{100}{75}=1600$
Therefore, required number is $1600$.
Now, $\displaystyle\frac{5}{8}$ of the number $=\displaystyle\frac{5}{8}\times 1600=1000$.

Thus the required number is $1000$.

Solve for $x$:

$35.453 =\dfrac{34.968x+ 36.956(100- x)}{100}$

  1. $7.56$

  2. $756$

  3. $75.6$

  4. $0.756$


Correct Option: C
Explanation:

$\\35.453\times 10=34.968x+36.956(100-x)\\ 3545.3=(34.968-36.956)x+3695.6\\\therefore x=(\frac{3545.3-3695.6}{34.968-36.956})\\ (\frac{-150.3}{-1.988})=75.6$

The terminating decimal expansion of the number $\dfrac{{337}}{{125}}$ is ........

  1. $2.666$

  2. $2.966$

  3. $2.696$

  4. $2.698$


Correct Option: C
Explanation:

$\dfrac{337}{125}=2.696$

Find the value of $(1.01)^{5}$ correct upto $3$ decimal places

  1. $1.015$

  2. $2.625$

  3. $1.651$

  4. $1.051$


Correct Option: D
Explanation:
${ \left( 1.01 \right)  }^{ 5 }$

$=1.01\times 1.01\times 1.01\times 1.01\times 1.01$

$=1.051$

The value of $\dfrac{8492 \times 3.72}{47.8 \times 52.24}$ is

  1. $1.265$

  2. $14.75$

  3. $1.475$

  4. $12.65$


Correct Option: A

$\displaystyle \frac{24.23\times 1.423\times 34.21}{521.3\times 413.32\times 2.53}$ is same is 

  1. $\displaystyle \frac{2423\times 1423\times 3421}{5213\times 41332\times 253}$

  2. $\displaystyle \frac{2423\times 1423\times 3421}{5213\times 4133.2\times 2.53}$

  3. $\displaystyle \frac{2.423\times 14.23\times 342.1}{521.3\times 4133.2\times 2.53}$

  4. $\displaystyle \frac{24.23\times 14.23\times 3.421}{5.213\times 41332\times 0.253}$


Correct Option: C
Explanation:

Option c is correct answer.

 
As in that expression total numbers after decimal is same as the given expression.


$\displaystyle \frac{24.23\times 1.423\times 34.21}{521.3\times 413.32\times 2.53}$  $=\displaystyle \frac{2.423\times 14.23\times 342.1}{521.3\times 4133.2\times 2.53}$

Simplify : $\displaystyle \frac{3.6\times 0.48\times 2.50}{0.12\times 0.09\times 0.5}$

  1. 80

  2. 800

  3. 8000

  4. 80,000


Correct Option: B
Explanation:

$\displaystyle \frac{3.6\times 0.48\times 2.50}{0.12\times 0.09\times 0.5}=\frac{36\times 48\times 250}{12\times 9\times 5}=800 $


No of decimal places in num and den being equal

Which number is equal to
$\left ( \displaystyle \frac {0.1}{0.01}\, +\, \displaystyle \frac {0.01}{0.1} \right )\, ?$

  1. $10.1$

  2. $1.10$

  3. $1.01$

  4. $10.01$


Correct Option: A
Explanation:

Let us first write the decimals as fractions as follows:


$0.1=\dfrac { 1 }{ 10 } \ 0.01=\dfrac { 1 }{ 100 }$ 

Now, the given expression $\dfrac { 0.1 }{ 0.01 } +\dfrac { 0.01 }{ 0.1 }$ can be solved as follows:
 
$\dfrac { 0.1 }{ 0.01 } +\dfrac { 0.01 }{ 0.1 } =\dfrac { \dfrac { 1 }{ 10 }  }{ \dfrac { 1 }{ 100 }  } +\dfrac { \dfrac { 1 }{ 100 }  }{ \dfrac { 1 }{ 10 }  } =\dfrac { \dfrac { 1 }{ 1 }  }{ \dfrac { 1 }{ 10 }  } +\dfrac { \dfrac { 1 }{ 10 }  }{ \dfrac { 1 }{ 1 }  } =10+\dfrac { 1 }{ 10 } =\dfrac { (10\times 10)+1 }{ 10 } =\dfrac { 100+1 }{ 10 } =\dfrac { 101 }{ 10 } =10.1$

Hence, $\dfrac { 0.1 }{ 0.01 } +\dfrac { 0.01 }{ 0.1 }=10.1$

Simplify : $\displaystyle \frac{0.2\times0.2+0.2\times0.02}{0.044}$

  1. $0.4$

  2. $0.2$

  3. $0.1$

  4. $1$


Correct Option: D
Explanation:

We will first convert the decimals into fraction in the given fraction and then solve it as follows:


$\dfrac { 0.2\times 0.2+0.2\times 0.02 }{ 0.044 } =\dfrac { \left( \dfrac { 2 }{ 10 } \times \dfrac { 2 }{ 10 }  \right) +\left( \dfrac { 2 }{ 10 } \times \dfrac { 2 }{ 100 }  \right)  }{ \dfrac { 44 }{ 1000 }  } =\dfrac { \dfrac { 4 }{ 100 } +\dfrac { 4 }{ 1000 }  }{ \dfrac { 44 }{ 1000 }  } =\dfrac { \dfrac { 40+4 }{ 1000 }  }{ \dfrac { 44 }{ 1000 }  } =\dfrac { \dfrac { 44 }{ 1000 }  }{ \dfrac { 44 }{ 1000 }  } =1$

Hence, $\dfrac { 0.2\times 0.2+0.2\times 0.02 }{ 0.044 } =1$

Evaluate the square root of $\displaystyle \frac{0.342\times 0.684}{0.000342\times 0.000171}$.

  1. $1000$

  2. $1500$

  3. $2000$

  4. $2500$


Correct Option: C
Explanation:

Consider the given fraction $\dfrac { 0.342\times 0.684 }{ 0.000342\times 0.000171 }$ and find the square root as follows:

 
$\sqrt { \dfrac { 0.342\times 0.684 }{ 0.000342\times 0.000171 }  } \ =\sqrt { \dfrac { 342\times 684 }{ 342\times 171 } \times \dfrac { 1000000\times 1000000 }{ 1000\times 1000 }  } \quad \quad \quad \quad \quad \quad \quad \left{ \because \quad 0.1=\dfrac { 1 }{ 10 }  \right} \ =\sqrt { 4\times 1000000 } \ =\sqrt { 4000000 } \ =\sqrt { { \left( 2000 \right)  }^{ 2 } } \ =2000$

Hence, the square root of $\dfrac { 0.342\times 0.684 }{ 0.000342\times 0.000171 }$ is $2000$.

The value of $\displaystyle \frac{3.157\times 4126\times 3.198}{63.972\times 2835.121}$ is closest to

  1. $0.002$

  2. $0.02$

  3. $0.2$

  4. $2$


Correct Option: C
Explanation:

The expression approximately =$\displaystyle \frac{3.2\times 4126\times 3.2}{64\times 2835}=0.232=0.2 (approx)$

A student was asked to simplify $\displaystyle \frac{0.6\times 0.6\times 0.6+0.5\times 0.5\times 0.5+0.1\times 0.1-0.09}{0.6\times 0.6+0.5\times 0.5+0.1\times 0.1-0.41}$ and his answer was 0.6 By what per cent was his answer wrong

  1. 25%

  2. 100%

  3. 50%

  4. 120%


Correct Option: C
Explanation:

$\displaystyle \frac{0.6\times 0.6\times 0.6+0.5\times 0.5\times

0.5+0.1\times 0.1-0.09}{0.6\times 0.6+0.5\times 0.5+0.1\times

0.1-0.41}$


$\dfrac{0.216+0.125+0.01-0.09}{0.36+0.25+0.01-0.41}= \dfrac{0.261}{0.21} = 1.2$

Student got answer as $0.6$

Hence His Answer was wrong with $\dfrac{0.6}{1.2}\times 100= 50$%

The square root of $\frac {(0.75)^3}{1-(0.75)}+(0.75+(0.75)^2+1)$ is

  1. 1

  2. 2

  3. 3

  4. 4


Correct Option: B
Explanation:

$\frac {(.75)^3+(1)^3-(.75)^3}{1-(0.75)}=\frac {1}{.25}=4$
square root $=2$

If x and y are positive numbers such that x + y = 1, which of the following could be the value of 100x + 200y?
I. 80
II. 140
III. 199

  1. II only

  2. III only

  3. I and II

  4. I and III

  5. II and III


Correct Option: E
Explanation:

Given x+y=1  So 100x+100y=100 and 200x+200y =200

So       100 < 100x + 200y <200
II and III satisfies this situation So correct answer will be Option E

Evaluate:


$\displaystyle \frac{(6.4)^{2}-(5.4)^{2}}{(8.9)^{2}+(8.9\times 2.2)+(1.1)^{2}}$

  1. $0.118$

  2. $0.112$

  3. $0.110$

  4. $0.104$


Correct Option: A
Explanation:

Given exp =$\displaystyle \frac{(6.4+5.4)(6.4-5.4)}{(8.9+1.1)^{2}}=\frac{11.8\times1 }{100}=0.118$

Simplify: $12.28 \times 1.5 - 36 \div 2.4$

  1. $3.24$

  2. $3.42$

  3. $4.32$

  4. $4.23$


Correct Option: B
Explanation:

$\displaystyle 12.28\times 1.5-\frac{36}{2.4}=18.42-15=3.42$

The value of the following is $\displaystyle \frac{(0.44)^{2}+(0.06)^{2}+(0.024)^{2}}{(0.044)^{2}+(0.006)^{2}+(0.0024)^{2}}$

  1. $0.100$

  2. $0.01$

  3. $100$

  4. $1$


Correct Option: C

$\displaystyle \frac{(0.22)^{3}+(0.11)^{3}+(0.32)^{3}}{(0.66)^{3}+(0.96)^{3}+(0.33)^{3}}-\frac{(0.32)^{3}+(0.45)^{3}-(0.77)^{3}}{81(0.32)(0.45)(0.77)}$ equals

  1. 1

  2. $\displaystyle \frac{1}{11}$

  3. 0

  4. -1


Correct Option: C
Explanation:

$\frac{(0.22)^{3}+(0.11)^{3}+(0.32)^{3}}{(0.66)^{3}+(0.96)^{3}-(0.33)^{3}}+\frac{(0.32)^{3}+(0.45)^{3}-(0.77)^{3}}{81(0.32)(0.45)(0.77)}$
$=\frac { 8(0.11)^{ 3 }+(0.11)^{ 3 }+(0.32)^{ 3 } }{ 216(0.11)^{ 3 }+27(0.32)^{ 3 }+27(0.11)^{ 3 } } -\frac { (0.32)^{ 3 }+(0.45)^{ 3 }-(0.32+0.45)^{ 3 } }{ 81(0.32)(0.45)(0.77) } $
$=\frac { 9(0.11)^{ 3 }+(0.32)^{ 3 } }{ 243(0.11)^{ 3 }+27(0.32)^{ 3 } } -\frac { (0.32)^{ 3 }+(0.45)^{ 3 }-(0.32+0.45)^{ 3 } }{ 81(0.32)(0.45)(0.77) } $
$=\frac { 9(0.11)^{ 3 }+(0.32)^{ 3 } }{ 27{ 9(0.11)^{ 3 }+(0.32)^{ 3 }}  } -\frac { (0.32)^{ 3 }+(0.45)^{ 3 }-[(0.32)^{ 3 }+(0.45)^{ 3 }+3(0.32)(0.45)(0.32)+(0.45) }{ { 81(0.32)(0.45)(0.77) } } $
$=\frac{1}{27}-\frac{1}{27}$
$=0$

What is the value of $(7.5 \times 7.5 + 37.5 + 2.5 \times 2.5) ?$

  1. $30$

  2. $60$

  3. $80$

  4. $100$


Correct Option: D
Explanation:

$(7.5 \times 7.5 + 37.5 + 2.5 \times 2.5) $

$=(7.5)^{2}+2\times 7.5\times 2.5+(2.5)^{2}$
$ =(7.5+2.5)^{2}$         ....[Using $a^2+2ab+b^2 = (a+b)^2]$
$=10^{2}$
$=100$

If k is an integer, and if $0.02468 \times 10^k$ is greater than 10,000, what is the least possible value of k?

  1. 7

  2. 4

  3. 6

  4. 5


Correct Option: C
Explanation:

Multiplying 0.02468 by a positive power of 10 will shift the decimal point to the right. Simply shift the decimal point to the right until the result is greater than 10,000. Keep track of how many times you shift the decimal point. Shifting the decimal point 5 times results in 2,468. This is still less than 10,000. Shifting one more place yields 24,680, which is greater than 10,000.

The value of $0.768 \times 0.768 - 2 \times 0.768 \times 0.568 + 0.568 \times 0.568$ is:

  1. $0.4$

  2. $0.04$

  3. $0.004$

  4. $0.0004$


Correct Option: B
Explanation:
Given,

$0.768 \times 0.768 - 2\times 0.768 \times 0.568 + 0.568 \times 0.568$


$= ( 0.768 )^2 - 2 \times  0.768 \times 0.568 + ( 0.568 )^2$

$Using\  identity \ \because { a^2 - 2 ab + b^2 = ( a - b )^2 }$

$= ( 0.768 - 0.568 )^2$

$= ( 0.2 )^2$

$= 0.04.$

$\therefore The\  option\ B\  is \ correct .$


The value of $\left( {0.3} \right)\left( {0.3} \right) - 2\left( {0.3} \right)\left( {0.2} \right) + \left( {0.2} \right)\left( {0.2} \right)$

  1. $0.1$

  2. $0.01$

  3. $1$

  4. $0.1 \times 0.1$


Correct Option: B,D
Explanation:
$0.3 \times 0.3 - 2\times 0.3 \times 0.2 + 0.2 \times 0.2$

$= ( 0.3 )^2 - 2 \times  0.3 \times 0.2 + ( 0.2 )^2$

$Using\  identity \ \because { a^2 - 2 ab + b^2 = ( a - b )^2 }$

$= ( 0.3 - 0.2 )^2$

$= ( 0.1)^2$

$= 0.1\times0.1$

$= 0.01.$

$\therefore The\  option\ B\ and\ D\ both \ are\  \ correct .$
- Hide questions